Complex Polynomial of nth degree

Click For Summary
For a polynomial P(z) of degree n (n ≥ 1), it can be shown that there exists a positive number R such that |P(z)| exceeds |a_n||z|^n/2 for all z with |z| > R. To approach this, one method involves dividing the polynomial by z^n, allowing the expression to be analyzed as |z| approaches infinity. It is suggested that for sufficiently large |z|, the dominant term a_n z^n will outweigh the constant term a_0, leading to the desired inequality. Additionally, rewriting the polynomial to isolate the leading term can clarify the relationship between the terms as |z| increases. This establishes the foundational understanding needed to prove the statement effectively.
Nathew

Homework Statement


Show that if
P(z)=a_0+a_1z+\cdots+a_nz^n
is a polynomial of degree n where n\geq1 then there exists some positive number R such that
|P(z)|>\frac{|a_n||z|^n}{2}
for each value of z such that |z|>R

Homework Equations


Not sure.

The Attempt at a Solution


I've tried dividing through by the nth power of z. That way I can somehow incorporate the R value somehow but I'm not exactly sure where to go from here.

Thanks!
 
Physics news on Phys.org
Nathew said:

Homework Statement


Show that if
P(z)=a_0+a_1z+\cdots+a_nz^n
is a polynomial of degree n where n\geq1 then there exists some positive number R such that
|P(z)|>\frac{|a_n||z|^n}{2}
for each value of z such that |z|>R

Homework Equations


Not sure.

The Attempt at a Solution


I've tried dividing through by the nth power of z. That way I can somehow incorporate the R value somehow but I'm not exactly sure where to go from here.

Thanks!

Maybe you could start by showing that for large enough z,

##|z|^n > |a_0|##

And, perhaps, rewrite the equation with everything but ##a_n z^n## on the LHS.

Can you see, without doing any algebra, why it's true?
 
Last edited:
Question: A clock's minute hand has length 4 and its hour hand has length 3. What is the distance between the tips at the moment when it is increasing most rapidly?(Putnam Exam Question) Answer: Making assumption that both the hands moves at constant angular velocities, the answer is ## \sqrt{7} .## But don't you think this assumption is somewhat doubtful and wrong?

Similar threads

  • · Replies 6 ·
Replies
6
Views
2K
  • · Replies 1 ·
Replies
1
Views
2K
  • · Replies 7 ·
Replies
7
Views
2K
  • · Replies 2 ·
Replies
2
Views
2K
  • · Replies 6 ·
Replies
6
Views
3K
  • · Replies 13 ·
Replies
13
Views
3K
Replies
9
Views
2K
Replies
3
Views
2K
  • · Replies 6 ·
Replies
6
Views
4K
  • · Replies 1 ·
Replies
1
Views
1K